3.Insight IAS CSP 2018 Test 03

September 13, 2017 | Author: sangu1434 | Category: Methane, Hybrid Vehicle, Taxes, Oath Of Office, Natural Gas
Share Embed Donate


Short Description

csp...

Description

INSIGHT TEST-3 Q.1- Non-Resident Indians (NRIs) can invest in India as per a regulation of Foreign Exchange Management (Transfer or issue of Security by a person resident outside India) Regulations, 2000. However, as per the regulation 1. NRIs cannot invest in shares or debentures. 2. NRIs may contribute foreign capital either by way of capital contribution Which of the above is/are correct? A. 1 only B. 2 only C. Both 1 and 2 D. None Your Answer : B Correct Answer : B Answer Justification : Justification: Statement 1: As per Regulations, NRIs may invest in shares or partly paid up shares of an Indian company under Foreign Direct Investment (FDI) Scheme subject to certain terms and conditions. Statement 2: NRIs may even acquire securities or units on a Stock Exchange in India on repatriation basis (i.e. they can take the investment back abroad) under the Portfolio Investment Scheme. NRIs even hold FCNR deposits in India banks that earn them interest. We will cover FCNR in details in coming tests. Q Source: DSM/SBS/KA (Release ID :169208) Q.2- Hambantota deep-sea port was recently see in news because A. Sri Lanka agreed to sell a strategic stake in this port to China. B. A Major oil spill here was treated by genetically engineered bacteria. C. India has signed a MoU with Seychelles to develop this port for defense related purposes. D. It will be used as an alternate transit to the Straits of Malacca. Your Answer : A Correct Answer : A Answer Justification : Background: Hambantota port is a deep-water port in the southern tip of Sri Lanka. It sits close to busy east-west shipping lanes connecting Europe and Asia. Under the 99-year lease agreement, China will invest over a billion dollars in the port and marinerelated activities for developing this port.

Learning: The port is expected to play a key role in China’s Belt and Road initiative (BRI), which will link ports and roads between China and Europe. It is also considered that, this deal gives an advantage to China in the bunkering business, which provides fuel to ships. India is apprehensive that the port is part of Chinese ‘string of pearls’ with an objective to surround India and dock its military vessels. Q Source: http://www.thehindu.com/news/international/sri-lanka-china-sign-11-bn-hambantota-port-deal/articl e19385932.ece Q.3 A ‘Task Force on Shell Companies’ under the Joint Chairmanship of Revenue Secretary and Secretary, Ministry of Corporate Affairs was constituted in February, 2017. What are ‘Shell Companies’? A. Oil companies that cause heavy ecological damage B. Fly by night operators C. A corporate entity without active business operations D. A company engaged in deep sea precious stones extraction Your Answer : C Correct Answer : C Answer Justification : Concept: Shell Company is a corporate entity without active business operations or significant assets. It can’t be asserted that shell corporations are illegal. They are deliberate financial arrangements by many big companies to avoid taxes without attracting legal actions. Tax avoidance is not illegal, though it is not desirable. Learning: But many shell companies park black money, carryout illegal transactions and sometimes act as facilitators of money laundering. Often, shell companies remain untraceable and happen to be the vehicle of choice for money launderers, bribe givers and takers, tax evaders and financiers of terrorism. Most of the shell companies are registered in tax havens like British Virgin Islands or Cayman Islands. Recently it was found that a single address hosted 75 firms and no employees! The taskforce Q.4- With reference to “flammable ice”, recently seen in news, consider the following statements: 1. It consists of methane trapped within water crystals. 2. It is another name for permafrost. 3. All of its reserves are trapped in plateaus at higher latitudes. 4. India is considering it as an alternative energy source. Select the correct answer using the codes below. A. 1, 3 and 4 only B. 2 and 3 only C. 1 and 4 only D. only

Your Answer : C Correct Answer : A Answer Justification : Justification: Statement 1: Flammable ice (also known as methane hydrate or methane clathrates) consists of methane trapped within water crystals. It is the world’s largest natural gas resource trapped beneath permafrost and ocean sediment where low temperature and moderate pressure combine to trap methane in this specific way. Statement 2: Permafrost appears on top of it. Permafrost is frozen chunks of ice that often contain carbon and volatile gases. Statement 3: China has successfully produced natural gas from methane hydrate, also known as “flammable ice”, in an experimental project in the South China Sea (SCS). Most of its reserves are located deep in ocean floors, especially continental shelves. Statement 4: India, USA, China and Canada all are looking at it as an alternative energy source. Learning: The methane hydrate is highly flammable and energy-intensive fuel as one cubic metre of the compound can releases about 160 cubic metres of gas. It can break down into water and methane after temperature is raised or pressure is lowered. Q Source: China’s South China sea ocean drilling programme Q.5 The Government is implementing an Ethanol Blended Petrol (EBP) Programme in India. Consider the following with reference to it. 1. The Government has introduced free market pricing mechanism for procurement of ethanol under EBP Programme. 2. The Government has allowed procurement of ethanol produced from cellulosic and lignocellulosic feedstock. Which of the above is/are correct? A. 1 only B. 2 only C. Both 1 and 2 D. None Your Answer : Correct Answer : B Answer Justification : Background: For the development of Biofuels, a National Policy on Biofuels has been announced in the year 2009. So, under the EBP Oil Marketing Companies sell ethanol blended petrol with percentage of ethanol up to 10% as per BIS specifications, depending upon its availability. Justification: Statement 1: The prices are administered so that the availability of ethanol can be improved by adequately incentivizing the producers. Statement 2: This is an eco-friendly way to convert ethanol from bio feed stock. Learning: In order to encourage production of Biodiesel in the country, the Government had also announced the Biodiesel Purchase Policy in October, 2005. The Government has also permitted the direct sale of Biodiesel (B-100) for blending with High Speed Diesel to all consumers, in accordance with the specified blending limits and the standards

specified by the BIS. Q Source: RM/RS- USQ1844 - LS (Release ID :169151) Q.6- With reference to Indian Green Building Council (IGBC), consider the following statements. 1. It is a body under Department of Urban Development. 2. It organizes the Green Building Congress. 3. It offers green building rating programmes. Select the correct answer using the codes below. A. 1 only B. 2 and 3 only C. 2 only D. 3 only Your Answer : A Correct Answer : B Answer Justification : Justification: The IGBC is part of the Confederation of Indian Industry (CII) formed in 2001. It offers a wide array of services including developing new green building rating programmes, green building training programmes and certification services. It also organises Green Building Congress, its annual flagship event on green buildings. It closely works with several State Governments, Central Government, World Green Building Council etc. Q Source: IGBC conference 2017 Q.7- Consider the following about Advance Pricing Agreements (APA) scheme. 1. It was introduced by the Income-tax (IT) Act. 2. It cannot be signed with individual companies. 3. It endeavors to provide certainty to taxpayers in the domain of transfer pricing. 4. It is not applicable to international transactions. Select the correct answer using the codes below. A. 1 and 3 only B. 2 and 4 only C. 2, 3 and 4 only D. 1, 3 and 4 only Your Answer : Correct Answer : A Answer Justification : Concept: Suppose two subsidiaries A and B of the same company X are located in two different tax regimes. Say A is subjected to more taxes, and B has to pay less tax. If company X transfers most of its profit from A to B, it will need to less taxes then, because B is taxed at lower rate. This is called transfer pricing, when profits are transferred to subsidiaries to reduce tax burden. Justification: Statement 2 and 3: It is signed with companies so that this tax evasion can be regulated. It also gives certainty to the tax department in terms of revenue collection.

Q Source: The Central Board of Direct Taxes (CBDT) has entered into 9 Unilateral Advance Pricing Agreements (UAPAs) with Indian taxpayers in July, 2017. Q.8- Consider the following about Jiyo Parsi scheme. 1. It is a Central Sector Scheme focused on increasing employment opportunities for Parsis in India. 2. It provides central assistance for expatriate Parsis to transfer their skills to native Parsis. Which of the above is/are correct? A. 1 only B. 2 only C. Both 1 and 2 D. None Your Answer : D Correct Answer : D Answer Justification : Justification: Its main objective is to reverse the declining trend of Parsi population by adopting a scientific protocol and structured interventions. It has two components: Medical Assistance and Advocacy (Counselling). The first phase of the scheme was launched initiated in 2013. Learning: Minorites are defined both at the Central and State level. Parsis form a minority community in India, and their welfare is the responsibility of the Central government. For a state declared minority, state governments provide for welfare plans. Q Source: The Union Ministry of Minority Affairs has launched Jiyo Parsi Publicity Phase-2 in Mumbai Q.9- ‘Mission Purpose’, ‘Breakthrough Starshot’ and ‘Sprites’ were recently seen in news in context of A. Interstellar missions B. Cost effective sea water desalination plants C. Autonomous self-driving cars D. Genetically engineered clonal species Your Answer : B Correct Answer : A Answer Justification : Learning: Starshot is a comprehensive space program launched under the US $100 million Breakthrough Initiatives, announced by Yuri Milner and Stephen Hawking to develop and launch practical interstellar space missions. The program aims to demonstrate proof of concept for light-propelled spacecraft that could fly at 20 per cent of light speed. Its main objective is to send one-gram chips to star systems beyond the solar system in search of extraterrestrial intelligence. Mission Purpose was designed to test the performance of the ‘Sprites’ (tiniest satellite) electronics when in orbit. It will also explore the concept of solar sail propulsion in which spacecraft can be powered using only the sun’s radiation.

Sprites have been developed by researchers at Cornell University. Each of the mini Sprite spacecrafts are built on a single 3.5*3.5 cm circuit board. They weigh just four grams each. These tiny satellites also mark the next step in the field of spacecraft miniaturization that can contribute to the development of centimeter and gram-scale StarChips envisioned under the Breakthrough Starshot project. Q Source: Often in news Q.10- Muntra, recently rolled out by the Defense Research and Development Organization (DRDO) is a/an A. Unmanned remotely operated tank B. Stealth land mine C. Indigenously developed fifth generation aircraft D. Airborne Radar system for fighter planes Your Answer : Correct Answer : A Answer Justification : Learning: It is India’s first unmanned, remotely operated tank at its Chennai lab. It can be used for surveillance, mine detection and reconnaissance in areas with nuclear and biological threats. It is also likely to be used in Naxal-hit areas. Q Source: Defense related updates 11 ‘Sagar Vani’ system released by the Union Ministry of Earth Science is A. An integrated information dissemination system that will serve the coastal community regarding their safety B. A SONAR system for deep sea mineral exploration C. A language code for Indian Navy to communicate more securely D. None of the above Your Answer : C Correct Answer : A Answer Justification : Learning: It serves the coastal community, especially the fishermen community with the advisories and alerts towards their livelihood as well as their safety at Sea. It is a software platform where various dissemination modes will be integrated on a single central server. It includes Multi Lingual SMS, Voice Call/Audio Advisory, Social Media (Facebook, Twitter, etc.) etc. Application: It can disseminate services in local languages using advanced artificial intelligence (AI) and machine learning capabilities. This ‘Sagar Vani’ system compares with the most advanced countries’ services in terms of speed of delivery, diverseness of services and omni channel capabilities. The system also has facility to provide access to various stakeholders (State Fishery Departments, Disaster Management Authorities, NGOs etc.) so that they will be able to further disseminate these ocean information and alerts to the user community. Q Source: Initiative of Ministry of Earth Sciences: PIB 12 With reference to the World Monuments Fund (WMF), consider the following:

1. It is a dedicated fund under UNESCO managed by the affiliates of UNESCO. 2. Every member country of UNESCO has agreed to contribute to this fund. 3. Every year, WMF presents the Hadrian Award to international leaders who have advanced the preservation of world architecture. Select the correct answer using the codes below. A. 1 only B. 3 only C. 2 and 3 only D. 2 only Your Answer : A Correct Answer : B Answer Justification : Justification: Statement 1: WMF is a private, international, non-profit organization dedicated to the preservation of historic architecture and cultural heritage sites around the world. Statement 2: Founded in 1965, through donations and matching funds, WMF has worked with local community and government partners worldwide to safeguard and conserve places of historic value for future generations. To date, WMF has worked in more than 91 countries, including many UNESCO World Heritage Sites. Statement 3: This applies both for preservation of world art and architecture. Q Source: Additional Research: http://www.thehindu.com/news/international/jesuss-tomb-to-be-unveiled-to-public/article17566017.e ce 13 With reference to the technology, SurfNet, that was recently seen in news, consider the following: 1. It is an artificial intelligence (AI) based software. 2. It can construct 3D models from 2D shapes. 3. It is presently being used for printing tissues and organelles of human beings. Select the correct answer using the codes below. A. 1 and 2 only B. 2 and 3 only C. 3 only D. 2 only Your Answer : Correct Answer : A Answer Justification : Justification: Statement 1: SurfNet software utilizes machine learning to analyze 2D shapes and convert them into projected 3D forms. In an advance version, it can allow future robots to navigate in the real world. In addition to transforming 2D shapes, the technology can also merge two shapes into one another. Statement 3: They were already being printed. Also, this is only a software, not a 3D printer. self-driving cars. SurfNet will give them ability to understand the 3D environment around them using standard 2D cameras.

It can also be used to create 3D content for virtual reality (VR) and augmented reality (AR) by simply using standard 2D photos. Q Source: Breakthrough initiative has been in news for a long time 14 Who administers the oath of the President of India? A. Vice-President B. Prime Minister of India C. Speaker, Lok Sabha D. Chief Justice of India Your Answer : D Correct Answer : D Answer Justification : Learning: According to Article 60 of the Constitution of India, every President and every person acting as President or discharging the functions of the President before entering upon office shall make and subscribe oath to the office. The oath inter alia includes the oath to preserve, protect and defend the constitution. In this sense, President takes oath as guardian of Indian Constitution. The oath is administered by Chief Justice of India or in his absence by the senior-most Judge of the Supreme Court available. Q Source: Presidential Oath: July 2017 15 The Petrapole-Benapole Integrated Check Post is a major route for A. India-Bangladesh bilateral trade B. India-Bhutan border trade C. India-China transit trade D. India-Pakistan Wagha border trade Your Answer : Correct Answer : A Answer Justification : Learning: In order to facilitate movement of cargo across the border, India and Bangladesh have recently agreed to operate Petrapole-Benapole ICP 24x7. The 24x7 operationalization of Petrapole-Benapole ICP is expected to be a significant milestone towards expeditious clearance of cargo and, hence boost the bilateral trade between the two countries. Land Ports Authority of India (LPAI) and Central Board of Excise and Customs (CBEC) are the implementing agencies for this agreement from the Indian side. Q Source: MJPS (Release ID :169332) 16 With reference to the Unorganised Workers' Social Security Act 2008, consider the following: 1. It covers self-employed workers as well as daily-wage workers. 2. It provides for constitution of National Social Security Board. 3. It issues a universal labour identification number (ULIN) to each labour in the unorganized sector. Select the correct answer using the codes below. A. 1 and 2 only B. 3 only C. 2 and 3 only D. 1 only

Your Answer : C Correct Answer : A Answer Justification : Justification: Statement 1: It is for the welfare of the unorganised workers (meaning home-based workers, self-employed workers or daily-wage workers). Statement 2: The National Social Security Board at the Central level shall recommend formulation of social security schemes viz life and disability cover, health and maternity benefits, old age protection and any other benefit as may be determined by the Government for unorganised workers. Statement 3: LIN has been recently issued under Shramev Jayate Yojana since 2015. It wasn’t under this act. Q Source: AK (Release ID :169273) 17 With reference to manufacture of firecrackers in India, consider the following: 1. It is mandatory for firecracker manufacturers to mention the composition of explosives and their quantity on the packet. 2. It is not mandatory to mention the consequential effects of fireworks on the environment and human health on the label of the packet. Which of the above is/are correct? A. 1 only B. 2 only C. Both 1 and 2 D. None Your Answer : A Correct Answer : C Answer Justification : Justification: Statement 1: This is as per the Explosives Rules, 2008. Statement 2: However, it is not mandatory to mention the consequential effects of fireworks on the environment and human health on the label of the packet or carton. This may be because the effects are well known or that these products aren’t sold like general consumer products. Their sale can be regulated by the district administration (for e.g. by providing an isolated ground for sale of firecrackers). Q Source: MJPS (Release ID :169309) 18 As per extant FDI policy in India 1. Foreign investment up to 100 percent is permitted for a company engaged in the business of manufacturing mobile phones. 2. There is a 30 per cent local sourcing of component for any FDI related manufacturing activity undertaken in India. Which of the above is/are correct? A. 1 only B. 2 only C. Both 1 and 2 D. None Your Answer : D

Correct Answer : A Answer Justification : Justification: Statement 1: This is under the automatic route where investment in the country can be made without prior approval either of the Government or the Reserve Bank of India. FDI is largely a matter of private business decisions, and therefore, no such details regarding efforts made by foreign companies to set up for e.g. Apple i-phone facilities are required. Statement 2: This isn’t applicable to all manufacturing units, for e.g. it is a restriction for mobile phone manufacturing units in India. Recently Apple submitted a representation to waive off this requirement because it would affect their profit margins. However, in case of ‘state-of-art’ and ‘cutting-edge technology’, sourcing norms can be relaxed subject to Government approval. This relaxation will be valid for an initial period of three years from the opening of first store, and thereafter, such entities will be required to meet the domestic sourcing norms. Q Source: MJPS (Release ID :169307) 19 Archaeological Sites are often preserved and cleaned using several chemical conservation approaches by ASI. Which of these come under such approaches? 1. Using a mixture of moss and lichens on monuments 2. Adding Suspended Particulate Matter (SPM) to clean art pieces 3. Removal of calcareous deposits and accretions 4. Bio-cide treatment Select the correct answer using the codes below. A. 1, 2 and 3 only B. 3 and 4 only C. 2, 3 and 4 only D. 1 and 4 only Your Answer : Correct Answer : B Answer Justification : Justification: Statement 1: There are certain biological factors such as Growth of Moss, Lichen, Algae, Fungi and higher plants on a monument. They not only impart a ugly look to the monument but also cause physical & chemical damaged to the building material. So, 1 is incorrect. Statement 2: Suspended Particulate Matter (SPM) and other chemically active pollutant species along with dust results in disfiguring of the monument. Similarly, variation in climatic conditions, moisture and intense solar radiation are also responsible for the decay of specific building materials. Statement 3 and 4: General cleaning of monuments, mud pack cleaning, removal of calcareous deposits and accretions, bio-cide treatment, consolidation and water repellent treatment, are some of the known ways of chemical treatment. Learning: Ancient Monuments and Archaeological Sites and Remains Act – 2010 provides for the preservation of ancient and historical monuments, archaeological sites, and remains of national importance and also provides penalty against acts of human vandalism. Other methods of cleaning: Mud Pack Cleaning is used for the cleaning of plain and decorated

marble surface. This method is being used successfully for the conservation of Taj Mahal and other marble structures. Very dilute mixture of ammonia solution and a non ionic detergent is used for the easy removal of accretions and biological growth with mild brushing. Bleaching powder slurry in aqueous medium is used to remove micro-vegetation from lime plastered surface. 20 Consider the following about India Brand Equity Foundation (IBEF) and associated provisions. 1. It is a non-profit NGO certifying khadi and handloom products. 2. Its objective is to promote and create international awareness of the ‘Made in India’ label in markets overseas. 3. India’s Foreign Trade Policy Statement released in 2015 emphasises the need of a branding strategy. 4. Activities of IBEF are limited to plantations, textiles and leather sectors. Select the correct answer using the codes below. A. 2 and 3 only B. 1 and 4 only C. 1, 2 and 4 only D. 1, 2 and 3 only Your Answer : Correct Answer : A Answer Justification : Justification: Statement 1: It is a trust established by the Department of Commerce, Ministry of Commerce and Industry. Statement 2: IBEF's primary objective is to promote and create international awareness of the Made in India label in markets overseas and to facilitate dissemination of knowledge of Indian products and services. Towards this objective, IBEF works closely with stakeholders across government and industry Statement 3: This is to promote Indian exports as well as push Make in India campaign. Statement 4: IBEF has undertaken focused branding activities for sectors namely engineering, pharma, plantations (tea, coffee and spices), services, textiles and leather. Q Source: MJPS (Release ID :169316) 21 The Government has launched a new scheme namely “Financial Support to MSMEs in ZED Certification Scheme”. The objective(s) of the scheme includes 1. Promote adaptation of energy efficient manufacturing 2. Drive manufacturing with adoption of Zero Defect production processes and without impacting the environment. 3. Establish a MSME Product Certifying Authority (MPCA) to assess quality of their products 4. Support ‘Make in India’ campaign. Select the correct answer using the codes below. A. 1 and 3 only B. 2 and 4 only C. 1, 2 and 4 only D. 1, 2, 3 and 4

Your Answer : Correct Answer : C Answer Justification : Justification: ZED strives for quality manufacturing. The objective of the scheme for promotion of ZED manufacturing amongst MSMEs and ZED Assessment for their certification so as to (these are other than what is mentioned in the question): Develop an Ecosystem for Zero Defect Manufacturing in MSMEs. Encourage MSMEs to constantly upgrade their quality standards in products and processes. Develop professionals in the area of ZED manufacturing and certification. Q Source: AK/RM (Release ID :169325) 22 Ministry of Tourism has released Guidelines for promotion of Medical and Wellness Tourism as Niche Tourism Product. As per the guidelines 1. Ministry of Tourism offers financial support to accredited Medical and Wellness Tourism Service Providers 2. Medical tourists from any country shall be provided Visa on arrival and e-visa facilities Which of the above is/are correct? B. 2 only C. Both 1 and 2 D. None Your Answer : B Correct Answer : A Answer Justification : Justification: Tourism products including medical and wellness tourism are mostly driven by the private sector and cost of such services are determined by market forces. However, the Ministry of Tourism is taking steps to promote this from the side of government as well. The grant is for Marketing Development Assistance, for Publicity, organising workshop/events/seminars and for organising Wellness and Medical Tourism Promotion shows. Q Source: Sanjay Kumar/jk/Parl. No. 1/26-07-2017: (Release ID :169061) 23 The VAJRA scheme was launched by the Science and Engineering Research Board (SERB) to A. Connect the Indian academic and research and development (R&D) systems to the best of global science and scientists B. Develop anti-neurotoxins that can be used to neutralize agents used in a potential chemical warfare C. Provide mobile air-defence guards to marine ships used as warships D. Increase participation from local ground level innovations to mainstream science and technology Your Answer : Correct Answer : A Answer Justification : Learning: SERB is a statutory body of the Department of Science and Technology (DST). It recently launched a ‘Visiting Advanced Joint Research (VAJRA) Faculty Scheme’ for a sustained international collaborative research.

The scheme offers adjunct / visiting faculty assignments to overseas scientists, faculty members and R&D professionals including NRI and OCI to undertake high quality collaborative research in cutting edge areas of science and technology. This is to be done with one or more Indian collaborators of public funded academic and research institutions of India. Q Source: RDS/nb (Release ID :169081) 24 What is the importance of Kanchi in context of Ancient India? 1. Ghatika at Kanchi was an ancient centre of leaning attracting students from India and abroad. 2. Founder of the Kadamba dynasty, Mayurasarman, studied Vedas at Kanchi. 3. Dharmapala, who later became the Head of the Ancient Nalanda University, belonged to Kanchi. 4. It was known as the religious capital of the Southern India. 5. Kanchi hosts Kailasanatha which is one of the largest and most ornate ancient temples in the whole of India. Select the correct answer using the codes below. A. 2 and 5 only B. only C. only D. 1, 2, 3, 4 and 5 Your Answer : Correct Answer : D Answer Justification : Justification: Once a capital of the Pallava dynasty, Kanchipuram was also a noted centre of learning for Tamil and Sanskrit scholars. Dinganaga, a Buddhist writer came to study at Kanchi. It hosts many temples dedicated to Shiva and Vishnu. Kanchi was also the home of the famous 6th century CE poet Bharavi who wrote the Kiratarjuniya and the famous 11th to 12th century CE Hindu philosopher Ramanuja. Still today an important religious centre, the site has over hundred temples and is also noted for its production of fine silk saris. Q Source: Additional Research: Page 118: TN 11th Standard Textbook 25 Pancha Siddhantika, a work of Varahamihira, deals with A. Geographical domains of India B. Astronomical systems C. Occult and mysticism D. Principles of social living Your Answer : B Correct Answer : B Answer Justification : Learning: He was also a great authority on astrology. In this context, he composed Pancha Siddhantika, about the five astronomical systems. Along with Aryabhatta, his contribution is considered seminal in Ancient India. His work Brihadsamhita is a great work in Sanskrit literature. It deals with a variety of subjects like astronomy, astrology, geography, architecture, weather, animals, marriage and omens. His Brihadjataka is considered to be a standard work on astrology. Q Source: Page 108: TN 11th Standard Textbook

26 In India, the safety aspects of genetically modified crops are assessed by which of these bodies? 1. Institutional Biosafety Committees (IBSCs) 2. Review Committee on Genetic Manipulation (RCGM) 3. Genetic Engineering Appraisal Committee (GEAC) 4. National Biopiracy authority (NBA) Select the correct answer using the codes below. A. 1, 2 and 3 only B. 2 and 4 only C. 1 and 3 only D. 2, 3 and 4 only Your Answer : A Correct Answer : A Answer Justification : Background: GEAC is the apex body constituted in the Ministry of Environment and Forests. It was set up under 'Rules for Manufacture, Use, Import, Export and Storage of Hazardous Microorganisms/Genetically Engineered Organisms or Cells 1989', under the Environment Protection Act, 1986. The Rules of 1989 also define five competent authorities i.e. IBSC, RCGM, GEAC, State Biotechnology Coordination Committee (SBCC) and District Level Committee (DLC) for handling of various aspects of the rules. Justification: Statement 1: Its main function is to note, examine and approve proposals involving rDNA work; to ensure adherence of r-DNA Safety Guidelines- 1990 of Government and inspection of containment facilities at R&D and production units. Statement 2: Main functions of RGCM are: To bring out manuals of guidelines specifying producers for regulatory process on GMOs in research, use and applications including in industry with a view to ensure environmental safety. To review all on going r-DNA projects involving high risk category and controlled field experiments To lay down producers for restriction or prohibition, production, sale, import & use of GMOs both for research and applications. To authorize imports of GMOs/ transgenes/ transgenic seeds for research purposes. Q Source: RDS/nb (Release ID :169320) 27 With reference to Ancient India, who introduced the Pattini cult in Tamilnadu? A. Cheran Senguttuvan B. Rajaraja II C. Pulakesin I D. Narasinghvarmana Your Answer : Correct Answer : A Answer Justification : Justification: He belonged to 2nd century A.D. Senguttuvan introduced the Pattini cult or the worship of Kannagi as the ideal wife in Tamil Nadu.

The stone for making the idol of Kannagi was brought by him after his Himalayan expedition. The consecration ceremony was attended by many princes including Gajabhagu II from Sri Lanka. Learning: His younger brother was Elango Adigal, the author of Silappathigaram. Among his military achievements, his expedition to the Himalayas was remarkable. He defeated many north Indian monarchs. Q Source: Page 87: TN 11th Standard Textbook 28 Family Participatory Care (FPC) as an important concept of health care mainly focuses on A. Newborn healthcare B. Geriatric population C. Urban sanitation and hygiene D. Health insurance in India Your Answer : A Correct Answer : A Answer Justification : Learning: Sick and newborn are highly vulnerable and require careful nurturing in order to survive the neonatal period and first year of life. This approach provides for partnership between health care staff and families in care of sick newborns admitted in the Special Newborn Care Units (SNCU). Under it, the capacities of parents-attendants are built in newborn care through a structured training programme. MoHFW has recently released Operational Guidelines on FPC. The guidelines will be shared with the states for implementation to further improve the quality of care provided in the SNCUs across the country. Q Source: http://pib.nic.in/newsite/PrintRelease.aspx?relid=169187 29 The NITI Aayog has approved several high-tech mas rapid transport proposals to improve public transport. Consider the following about them. 1. Hyperloop is a high air pressure tube connecting cities at speeds matching that of an aircraft. 2. Freight rail roads will be elevated corridors with rail lines where freight trucks can move on rails at high speed. 3. Pod taxis are underground transit systems that run entirely on solar power. 4. Hybrid electric buses combine a conventional internal combustion engine with an electric propulsion system. Select the correct answer using the codes below. A. 1 and 3 only B. 2 and 4 only C. 2, 3 and 4 only D. 1 and 2 only Your Answer : Correct Answer : B Answer Justification : Justification: This proposal includes high-tech mass rapid transportation technologies such as metrino, hyperloop, pod taxis, stadler buses, hybrid buses and freight rail road. Statement 1: They are near-vacuum tubes so that air friction in travel can be reduced.

Statement 2: A dedicated corridor allows for greater efficiency and speed. Statement 3: These are usually cabled taxis running on an elevated drive train. Statement 4: The addition of a battery-powered electric motor increases the fuel efficiency of hybrids in a number of ways. Idle-off feature turns off the car's conventional engine when the vehicle is stopped, saving fuel. The battery provides energy for the air conditioner and accessories while the vehicle idles at stoplights or in traffic, and the electric motor can start the vehicle moving again. If needed, the conventional engine will reengage to provide more power for acceleration. Learning: "Regenerative braking" is another fuel-saving feature. Conventional cars rely entirely on friction brakes to slow down, dissipating the vehicle's kinetic energy as heat. Regenerative braking allows some of that energy to be captured, turned into electricity, and stored in the batteries. This stored electricity can later be used to run the motor and accelerate the vehicle. Q Source: http://www.livemint.com/Politics/8fnjxsUr13noU98kvkPerO/Transport-ministry-to-explore-6-new-ma ss-rapid-transportatio.html 30 Aadhaar is presently being used as an identifier in respect of following school education related schemes? 1. Mid-Day Meal Scheme (MDMS) 2. Sarva Shiksha Abhiyan (SSA) 3. Rashtriya Madhyamik Shiksha Abhiyan (RMSA) 4. Inclusive Education for Disabled at Secondary Stage (IEDSS) Select the correct answer using the codes below. A. 1 and 2 only B. 2, 3 and 4 only C. 3 and 4 only D. 1, 2, 3 and 4 Your Answer : D Correct Answer : D Answer Justification : Justification: This is to reduce ghost beneficiaries, leakages in distribution of provisions. Also, this enforces accountability, transparency and efficiency in the governance of the schemes. Other covered schemes are: National Scheme of Incentive to Girls for Secondary Education (NSIGSE) National Means-cum-Merit Scholarship Scheme (NMMSS) Centrally Sponsored Scheme for providing quality education in Madrasa (SPQEM) Q Source: GG/AK/RK/Aadhaar for Social Welfare: (Release ID :169280) 31 he Satavahanas patronized Buddhism and Brahmanism. The evidence of this is clear from the fact that 1. They built several chaityas and viharas. 2. They prohibited Buddhist monks from acquiring land to help in their spiritual pursuit. 3. They persecuted Brahmanism. 4. They patronized the Prakrit language and literature.

Select the correct answer using the codes below. A. 1, 2 and 3 only B. 1 and 4 only C. 1, 3 and 4 only D. 2 and 3 only Your Answer : B Correct Answer : B Answer Justification : Justification: Statement 1: Vashishtaputra Pulamayi repaired the old Amaravathi stupa. Their architecture in Nagarjunakonda was also notable. Statement 2: They also made grants of villages and lands to Buddhist monks. Statement 3: Brahmanism was revived by the Satavahanas along with the performance of asvamedha and rajasuya sacrifices. Statement 4: Hala’s Sattasai is an excellent piece of Prakrit literature. Q Source: Page 77: TN 11th Standard Textbook 32 With reference to Ancient India, in the northern part of the country, the village headman was known as the A. Gramabhojaka B. Agrahara C. Dhammsurna D. Sentokwara Your Answer : A Correct Answer : A Answer Justification : Learning: Usually, men from the same family held the position for generations or via hereditary. The grama bhojaka was often the largest landowner. Generally, he had slaves and hired workers to cultivate the land. Besides, as he was powerful, the king often used him to collect taxes from the village. He also functioned as a judge, and sometimes as a policeman. Apart from the gramabhojaka, there were other independent farmers, known as grihapatis, most of whom were smaller landowners. Q Source: Revision Previous tests: Chapter 7: 6th NCERT History 33 Sangam poems mention the term muvendar. It was used for A. Land allotted to temples by Kings B. Administrative arrangements made to felicitate scholars of Tamil C. Heads of three ruling dynasties of Southern India D. Classical commentary on Vedas Your Answer : C Correct Answer : C Answer Justification : Learning: This is a Tamil word meaning three chiefs, used for the heads of three ruling families, the Cholas, Cheras, and Pandyas, who became powerful in south India around 2300 years ago. Each of the three chiefs had two centres of power: one inland, and one on the coast. Of these six cities, two were very important: Puhar or Kaveripattinam, the port of the Cholas, and

Madurai, the capital of the Pandyas. Q Source: Additional Research: Page 139: TN 11th Standard Textbook 34 Which of these developments can be attributed to Later Vedic age, as against the practice in the Vedic age? 1. Women lost their political rights of attending assemblies. 2. Child marriage were banned and the practice of sati was absent. 3. Women were not allowed to wear garments made of cotton. Select the correct answer using the codes below. A. 1 only B. 2 and 3 only C. 3 only D. 2 only Your Answer : A Correct Answer : A Answer Justification : Background: The Rig Vedic society was patriarchal. But, the condition of women was much better in this period that in the Later vedic age. Women were given equal opportunities as men for their spiritual and intellectual development. There were women poets like Apala, Viswavara, Ghosa and Lopamudra during the Rig Vedic period. Women could even attend the popular assemblies. There was no child marriage and the practice of sati was absent. Justification: Statement 1: In the family, the power of the father increased during the Later Vedic period. There was no improvement in the status of women. They were still considered inferior and subordinate to men. Women also lost their political rights of attending assemblies. Statement 2: Child marriages had become common during the later vedic period. Statement 3: In both periods, men and women wore upper and lower garments made of cotton and wool. A variety of ornaments were used by both men and women. Q Source: Revision Previous tests: Page 29: TN 11th Standard Textbook 35 With reference to Ancient texts, consider the following statements. 1. The Brahmanas are the law texts under Manusmriti that helped establish the predominance of Brahmana community. 2. Upanishads are treatises relating to prayer and sacrificial ceremony. 3. Aranyakas are called forest books and they deal with mysticism. 4. Yajurveda deals with music, chanting and medicine. Select the correct answer using the codes below. A. 1 and 2 only B. 3 only C. 1, 3 and 4 only D. 2 and 4 only Your Answer : B Correct Answer : B

Answer Justification : Justification: Besides the Vedas, there are other sacred works like the Brahmana, the Upanishads, the Aranyakas and the epics Ramayana and Mahabharata. The Brahmanas are the treatises relating to prayer and sacrificial ceremony. The Upanishads are philosophical texts dealing with topic like the soul, the absolute, the origin of the world and the mysteries of nature. The Aranyakas are called forest books and they deal with mysticism, rites, rituals and sacrifices. Yajurveda is a compilation of ritual offering formulas that were said by a priest while an individual performed ritual actions such as those before the yajna fire. Q Source: Revision Previous tests: Page 28: TN 11th Standard Textbook 36 Irula people 1. Are a tribe of the Andaman and Nicobar Islands 2. Practice iron smelting as the main occupation 3. Do not speak any known dialect Select the correct answer using the codes below. A. 1 and 2 only B. 3 only C. 2 only D. None of the above Your Answer : Correct Answer : D Answer Justification : Justification: Statement 1: Irula is an ethnic group of India. They inhabit the area of the Nilgiri mountains, in the states of Tamil Nadu and Kerala. Statement 2: Traditionally, the main occupation of the Irulas has been snake and rat catching. They also work as labourers (coolies) in the fields of the landlords during the sowing and harvesting seasons or in the rice mills. Fishing is also a major occupation. Statement 3: People of Irula ethnicity are called Irular, and speak Irula, which belongs to the Dravidian family. Unlike the Negrito tribes in the Andaman Islands who have retained their language, Irular speak the Irula language, a Dravidian language that is closely related to Tamil, Yerukala, Sholaga and other Tamil languages. Q Source: Additional Research: Page 90: TN 11th Standard Textbook 37 Buddhamitra was known for 1. Writing virasoliyam which is a work on Tamil Grammar 2. Erecting inscriptions on images of bodhisattvas and the Buddha near Ganges river Which of the above is/are correct? A. 1 only B. 2 only C. Both 1 and 2 D. None Correct Answer : C Answer Justification : Justification: Buddhamitra was a Buddhist nun from India during the Kushan Empire.

Statement 1: She is remembered because of dated inscriptions on images of bodhisattvas and the Buddha that she erected in three cities near the Ganges river. They mark her success in attracting money and patronage to the Sarvastivada, the sect of Buddhism to which she belonged. Statement 2: Virasoliyam attempts to find synthesis between Sanskrit and Tamil grammar. There were other books written on Tamil grammar as well like Yapperungalam and Yapperungalakkarigai by the Jain ascetic Amirtasagara. Q Source: Additional Research: Page 150: TN 11th Standard Textbook 38 Adipurana written by Gunabhadra is a work on the A. Commentary of esoteric aspects found in the Vedas B. Life stories of various Jain saints C. Sanskrit Grammar D. Verses used in sacrificial ceremony Your Answer : A Correct Answer : B Answer Justification : Justification: It is a 9th century Sanskrit poem composed by Jinasena, a Digambara monk. It deals with the life of Rishabhanatha, the first Tirthankara, and other saints. Learning: Amogavarsha I, who was a Jain patronized many Jain scholars. His teacher Jinasena composed Parsvabhudaya, a biography of Parsva in verses. Sakatayana wrote the grammer work called Amogavritti. And, the great mathematician of this period, Viracharya was the author of Ganitasaram. Q Source: Page 137: TN 11th Standard Textbook 39 The images of Shiva represented in various forms like Nataraja, Gangadhara, Ardhanareesvara, Somaskanda and Trimurthi can be found in A. Elephanta caves B. Ajanta caves C. Karla caves D. Kanheri rock-cut caves Your Answer : A Correct Answer : A Answer Justification : Learning: Elephanta is an island near Bombay. It was originally called Sripuri. The Portuguese after seeing the large figure of an elephant named it Elephanta. The sculptural art of the Rashtrakutas reached its zenith in this place. There is a close similarity between the sculptures at Ellora and those in Elephanta. They might have been carved by the same craftsmen. The most imposing figure of this temple is Trimurthi. The sculpture is six metre high. It is said to represent the three aspects of Shiva as Creator, Preserver and Destroyer. Q Source: Page 138: TN 11th Standard Textbook 40 If you compare the socio-economic life of Cholas and Sangam age, what is/are the chief similarities in these periods with respect to the position of women in society? 1. The practice of Sati was present. 2. Devadasi system was abolished.

3. Women were not allowed to write or study literature. 4. Love marriage was forbidden. Select the correct answer using the codes below. A. 1 only B. 2, 3 and 4 only C. 3 and 4 only D. 1, 2 and 4 only Your Answer : A Correct Answer : A Answer Justification : Justification: Statement 1 and 4: Love marriage was a common practice in the Sangam age. Women were allowed to choose their life partners. However, the life of widows was miserable. The practice of Sati was prevalent in the higher strata of society. Even in Cholas, practice of ‘sati’ was prevalent among the royal families. Statement 2: The devadasi system or dancing girls attached to temples emerged during the Chola period. There is evidence that they were often subjected to harassment by priets and officials. Statement 3: Women poets like Avvaiyar, Nachchellaiyar, and Kakkaipadiniyar flourished in the sangam period and contributed to Tamil literature. Q Source: Page 91 and 149: TN 11th Standard Textbook 41 With reference to Chola Education and Literature, consider the following: 1. Temples and mathas served as the only educational centres. 2. Not only Vedas and Epics, but subjects like mathematics and medicine were also taught. 3. Endowment of lands was made to run educational centres. Select the correct answer using the codes below. A. 1 and 2 only B. 2 and 3 only C. 3 only D. 1, 2 and 3 Your Answer : A Correct Answer : B Answer Justification : Justification: Statement 1: During this period, besides the temples and mathas as educational centres, several educational institutions also flourished. The inscription at Ennayiram, Thirumukkudal and Thirubhuvanai provide details of the colleges existed in these places. Statement 2 and 3: The development of Tamil literature reached its peak during the Chola period. And a lot of it came from the contribution of education and literature to the enrichment of this culture. Mathematics and medicine studies showed that Kings encouraged logical and intellectual pursuits and were not centred entirely on religious studies. Q Source: Page 149: TN 11th Standard Textbook 42 Vanavaramban, Vanavan, Kuttuvan, Irumporai and Villavar were famous titles assumed by A. Chera kings B. Sangam poets

C. Thiruvalluvar, a tamil philosopher and poet D. Temple priests in Southern Indian dynasties Your Answer : D Correct Answer : A Answer Justification : Learning: Hereditary monarchy was the form of government during the Sangam period. The Chera kings assumed titles like Vanavaramban, Vanavan, Kuttuvan, Irumporai and Villavar, the Chola kings like Senni, Valavan and Killi and the Pandya kings Thennavar and Minavar. Each of the Sangam dynasties had a royal emblem – carp for the Pandyas, tiger for the Cholas and bow for the Cheras. Q Source: Page 89: TN 11th Standard Textbook 43 Consider the following statements with reference to the South Indian dynasties. 1. Arikkamedu was a seaport during the Sangam period. 2. Uraiyur was famous for pearls. 3. Lord Murugan was the primary deity of the Sangam age. Select the correct answer using the codes below. A. 1 only B. 2 and 3 only C. 3 only D. 1, 2 and 3 Your Answer : D Correct Answer : D Answer Justification : Justification: Statement 1: External trade was carried between South India and the Greek kingdoms. The port city of Puhar became an emporium of foreign trade, as big ships entered this port with precious goods. Other ports of commercial activity include Tondi, Musiri, Korkai, Arikkamedu and Marakkanam. Statement 2: Uraiyur was an ancient Chola city with a fortress and city wall on the southern banks of the river Kaveri. It was known for pearls. Statement 3: The worship of Murugan was having an ancient origin and the festivals relating to God Murugan was mentioned in the Sangam literature. He was honoured with six abodes known as Arupadai Veedu. Q Source: Page 92: TN 11th Standard Textbook 44 Consider the following about the visit of the famous Chinese traveller, Fahien. 1. He visited India during the reign of Chandragupta II. 2. He stayed for over twenty years in India. 3. He came to India by sea route. 4. His main purpose of visiting India was to study its political and social system. 5. He studied Sanskrit in Patliputra. Select the correct answer using the codes below. A. 2, 3 and 4 only B. 1, 2 and 5 only

C. 1 and 5 only D. 1, 2, 4 and 5 only Your Answer : B Correct Answer : C Answer Justification : Justification: Statement 1 and 2: Out of his nine years stay in India, he spent six years in the Gupta empire. Statement 3: He came to India by the land route through Khotan, Kashgar, Gandhara and Punjab. He visited Peshawar, Mathura, Kanauj, Sravasti, Kapilavastu, Kusinagara, Pataliputra, Kasi and Bodh Gaya among other places. He returned by the sea route, visiting on the way Ceylon and Java. Statement 4: The main purpose of his visit was to see the land of the Buddha and to collect Buddhist manuscripts from India. Statement 5: He stayed in Pataliputra for three years studying Sanskrit and copying Buddhist texts. Q Source: Page 102: TN 11th Standard Textbook 45 Hiuen Tsang portray the social life in the times of Harsha. Consider the following with reference to it. 1. He mentions that the Sudras were forbidden from agriculture. 2. As per his documentation, the dead could not be subjected to water burial. 3. Dowry system was practiced in the empire. Select the correct answer using the codes below. A. 1 only B. 2 and 3 only C. 3 only D. 1 and 2 only Your Answer : C Correct Answer : C Answer Justification : Justification: The fourfold division of the society – Brahmin, Kshatriya, Vysya and Sudra – was prevalent. Statement 1: The Brahmins were the privileged section of the society and they were given land grants by the kings. Hiuen Tsang mentions that the Sudras practiced agriculture. Statement 2: Hiuen Tsang mentions three ways of disposal of the dead – cremation, water burial and exposure in the woods. Statement 3: The position of women was not satisfactory. The institution of Swyamvara (the choice of choosing her husband) had declined. Remarriage of widows was not permitted, particularly among the higher castes. The system of dowry had also become common. The practice of sati was also prevalent. Learning: Trade and commerce had declined during Harsha’s period, which is is evident from the decline of trade centres, less number of coins, and slow activities of merchant guilds. The decline of trade in turn affected the handicrafts industry and agriculture. Since there was no large-scale demand for goods, the farmers began to produce only in a limited way. This led to the rise of self-sufficient village economy. Q Source: Page 117: TN 11th Standard Textbook 46 IFAW (International Fund for Animal Welfare)

1. is a non-profit organization 2. provides hands-on assistance to animals 3. campaigns to end commercial whaling 4. was founded by the World Wildlife Fund (WWF) Select the correct answer using the codes below. A. 1 and 4 only B. 2 and 3 only C. 1, 2 and 3 only D. 2 and 4 only Your Answer : C Correct Answer : C Answer Justification : Justification: The International Fund for Animal Welfare (IFAW) is one of the largest animal welfare and conservation charities in the world. The group's declared mission is to "rescue and protect animals around the world. The International Fund for Animal Welfare (IFAW) was founded by a small group of people in 1969, to stop the commercial hunt for seal pups in Canada. The International Tiger Day (also known as Global Tiger Day) is celebrated every year on 29 July to raise awareness for tiger conservation. The IFAW recently organized major events on International Tiger conservation day. You can read more about it here http://www.ifaw.org/united-states/about-ifaw Q Source: As mentioned above 47 Consider the following: 1. They developed Vesara style of architecture. 2. Their structural temples exist at Aihole, Badami and Pattadakal. 3. Their administration was highly centralized. The above most accurately refer to? A. Cholas B. Chalukyas C. Cheras D. Pandyas Your Answer : A Correct Answer : B Answer Justification : Justification: Administration: The Chalukya administration was highly centralized unlike that of the Pallavas and the Cholas. Village autonomy was absent under the Chalukyas. Architecture: The Chalukyas were great patrons of art. They developed the vesara style in the building of structural temples. However, the vesara style reached its culmination only under the Rashtrakutas and the Hoysalas. Cave temple architecture was also famous under the Chalukyas. Their cave temples are found in Ajanta, Ellora and Nasik. The best specimens of Chalukya paintings can be seen in the Badami. Q Source: Additional Research: Page 134: 11th TN Textbook 48 With reference to Ancient India, mahajanapada rulers imposed taxes on

A. Crops and crafts persons only B. Goods, crops, herders, hunter gatherers and crafts persons C. Goods and royal services only D. Hunter gatherers and crafts persons only Your Answer : C Correct Answer : B Answer Justification : Background: As they were building huge forts and maintaining big armies, they needed more resources. Instead of depending on occasional gifts brought by people, as in the case of the raja of the janapadas, they started collecting regular taxes. Learning: Taxes on crops were the most important. This was because most people were farmers. There were taxes on crafts persons as well. These could have been in the form of labour. For example, a weaver or a smith may have had to work for a day every month for the king. Herders were also expected to pay taxes in the form of animals and animal produce. There were also taxes on goods that were bought and sold, through trade. And hunters and gatherers also had to provide forest produce to the raja. Q Source: Revision previous tests: Page 59-60: 6th NCERT 49 Dakshinapatha and Uttarapatha were A. Two great highways that have connected different parts of the sub-continent since the Iron Age B. Pali transcriptions of two different sects of Buddhism C. Extensions of Chola Empire in Sri Lanka and Northern India respectively D. The largest gopurams of Brihadeshwara temple made under Rajaraja I Your Answer : C Correct Answer : A Answer Justification : Learning: Dakshinapatha is a historical which has been used to describe the great southern highway in India, traveling from Magadha to Pratishthana. Uttarapatha or the great northern road ran from Taxila in Afganisthan, through the modern Punjab up to the western coast of Yamuna. Following the course of Yamuna, it went southwards up to Mathura, from there it passed on to Ujjain in Malwa and to Broach on western coast. The most important ruler of the Satavahanas, Gautamiputra Shri Satakarni and other Satavahana rulers were known as lords of the dakshinapatha. Q Source: Additional Research: Page 99: 11th TN Textbook 50 Consider the following about Traditional Knowledge Digital Library (TKDL), an Indian initiative. 1. It is to protect Indian traditional knowledge from exploitation by wrongful patents mainly at International Patent Offices. 2. It is maintained in a digitized format and is available only in old Sanskrit so as to avoid the misuse of ancient discoveries. Which of the above is/are correct? A. 1 only B. 2 only

C. Both 1 and 2 D. None Your Answer : A Correct Answer : A Answer Justification : Justification: It is an organisation of the Council of Scientific and Industrial Research. TKDL contains Indian traditional medicine knowledge available in public domain and pertains to classical/ traditional books related to Ayurveda, Unani and Siddha in a digitized format and is available in five international languages (English, French, German, Spanish and Japanese). India’s rich and time-tested traditional medicinal knowledge which exists in languages such as Sanskrit, Hindi, Arabic, Persian, Urdu, Tamil etc. is neither accessible nor comprehensible for patent examiners at the international patent offices. So, let’s say if some corporation patents the use of termuric in curing certain chronic diseases, it’ll amount to piracy of our traditional knowledge. If this information about ancient knowledge is already made available to patent offices for preverification in different languages beforehand, such misuse can be stopped. Q Source: http://www.thehindu.com/todays-paper/ancient-knowledge-hub-faces-grey-future/article17625761.e ce 51 Consider the following about the Senior Citizens Welfare Fund. 1. It was established along with the Central Social Welfare Board (CSWB). 2. It will be administered by an Inter-Ministerial Committee. 3. The fund shall not be audited by the CAG due to its fundamental character as a welfare fund. Select the correct answer using the codes below. A. 2 only B. 1 and 3 only C. 1 and 2 only D. 2 and 3 only Your Answer : Correct Answer : A Answer Justification : Justification: Statement 1: CSWF was established in the 1950s, so 1 is wrong. You can read about CSWB here http://www.cswb.gov.in/index1.asp?linkid=230&langid=1 The Centre brought in Senior Citizens’ Welfare Fund Act, 2015 (SCWF) as part of the Finance Act, 2015, which mandates transfer of unclaimed amounts of policyholders to the fund (SCWF) after a period of 10 years. Statement 2: The fund will be administered by an Inter-Ministerial Committee, headed by a Chairperson. The Committee will be competent to spend money from the fund for satisfying various objectives. Statement 3: The accounts of the fund will be open to audit by CAG, regularly. The Central Government will present the annual report and the one furnished by CAG to be laid before the Parliament. 52 National Anti-profiteering Authority will look after

A. Enforcing anti-trust or competitive laws in the industry on the lines of Competition Commission of India (CCI) B. Consumer grievances arising due to complaints raised by unreasonable pricing of goods and services C. Ensuring that benefits of a reduction in tax on supply of goods or services flow to the consumers D. None of the above Your Answer : Correct Answer : C Answer Justification : Concept: In the GST mechanism, the producers are provided an input tax credit. It works this way. Suppose, a producer requires INR 80 worth of cotton to produce a shirt, which is finally processed and priced at INR 100 in the market. In this process, the producer will have to pay taxes at two places – a) while purchasing the raw material, and b) while selling the final produce – shirt. This double taxation (tax on tax) is avoided by an input tax credit mechanism. Under this, the producers are compensated for any additional taxes they would have paid. Justification: When constituted by the GST Council, the National Anti-profiteering Authority shall be responsible for applying anti-profiteering measures in the event of a reduction in rate of GST on supply of goods or services. It will also check if the benefit of input tax credit is not passed on to the recipients by way of commensurate reduction in prices. Learning: In the event the National Anti-profiteering Authority confirms the necessity of applying anti-profiteering measures, it has the power to order the business concerned to reduce its prices or return the undue benefit availed along with interest to the recipient of the goods or services. If the undue benefit cannot be passed on to the recipient, it can be ordered to be deposited in the Consumer Welfare Fund. In extreme cases the National Anti-profiteering Authority can impose a penalty on the defaulting business entity and even order the cancellation of its registration under GST. Q Source: http://pib.nic.in/newsite/PrintRelease.aspx?relid=169000 53 Consider the following about electoral bonds that were introduced in the 216-17 annual budget. 1. They will be an interest-paying debt instrument. 2. They cannot be sold by banks. 3. They can be deposited in notified accounts of political parties. 4. The bonds will not carry the name of the donor. Select the correct answer using the codes below. A. 1 and 2 only B. 3 and 4 only C. 1, 2 and 3 only D. 2 and 3 only Your Answer : B Correct Answer : B Answer Justification : Concept: Electoral bonds are to be used for donations to political parties.

The advantage of the electoral bond system over donations through cheques is that many donors expressed reluctance to use cheques because it becomes transparent and leads to political vendetta by rivals. Features: As per the electoral bond mechanism announced in the Budget, the proposed bonds will resemble a promissory note and not an interest-paying debt instrument. They will be sold by authorised banks and can be deposited in notified accounts of political parties within the duration of their validity. The bonds will not carry the name of the donor and routing of the money through banks will ensure that only tax paid money comes into the political system. The government is planning to go ahead with electoral bonds even if consensus eludes on the issue as political parties have not come up yet with any suggestion on the proposal. Q Source: http://www.insightsonindia.com/2017/07/25/insights-daily-current-affairs-25-july-2017/ 54 Anti-dumping duty is usually applied in order to 1. Contain trade distortion 2. Increase imports of capital goods 3. Sterilize foreign capital inflows Select the correct answer using the codes below. A. 1 and 2 only B. 3 only C. 1 only D. 2 and 3 only Your Answer : C Correct Answer : C Answer Justification : Justification: Statement 1: Suppose a nation like China is dumping (exporting) its goods at exceptionally cheap prices so that it can capture the market. Since these cheap prices can’t be justified by market forces alone, the Indian government must apply a duty (like a tax) on the product to raise its price. This is to shield the domestic producers. Statement 2: Capital goods are usually not dumped by nations, since the industry is highly segmented and its hard to dump products at cheap prices. Statement 3: It has nothing to do with FDI or FII. Learning: India has initiated over two hundred anti-dumping investigations between 2012 and 2017 against various countries, including China and Indonesia. During the period, maximum number of cases were against China. Product categories on which the levy was imposed include chemicals, fibre boards, glass & glassware, pharmaceuticals and steel. Q Source: http://www.insightsonindia.com/2017/07/25/insights-daily-current-affairs-25-july-2017/ 55 If China were to takeover Doklam plateau, which region in or near India would be most affected by this move? A. Leh and Ladakh B. Siliguri Corridor C. India-Uttarakhand Border D. Sundarban delta Your Answer : B

Correct Answer : B Answer Justification : Learning: China is building a road project in the region which as per Bhutan would run from the town of Dokola to the Bhutanese army camp at Zompelri. This is seen by Bhutan as a direct violation of the agrements between two nations. India has said the Chinese road project threatens its access to the Siliguri corridor that connects our mainland to north-eastern states. The tense standoff between India and China over the Doklam plateau has only escalated, raising concerns in both capitals of an all-out military conflict. Q Source: http://www.insightsonindia.com/2017/07/25/insights-daily-current-affairs-25-july-2017/ 56 The Mekedatu dispute recently seen in news is between A. Karnataka and Tamil Nadu B. Government of India and Government of Sri Lanka C. Fishermen community from Kerala and Northern province of Sri Lanka D. Andhra Pradesh, Telangana and Karnataka Your Answer : Correct Answer : A Answer Justification : Learning: Karnataka intends to build a reservoir across river Cauvery near Mekedatu in Kanakapura taluk. It was first proposed along with Shivanasamudra hydro power project in 2003 with an intention to use the water for a hydro power station and supply drinking water to Bengaluru city. However, Tamil Nadu objected saying Karnataka had not sought prior permission for the project. Its argument was that the project would affect the flow of Cauvery water to Tamil : http://www.insightsonindia.com/2017/07/26/insights-daily-current-affairs-26-july-2017/ 57 Why the government has introduced the Central Road Fund (Amendment) Bill, 2017 in the Lok Sabha? A. To provide for development of national waterways (NWs) from this fund B. To add state and local highways to the provision list of this fund C. To upgrade road construction and maintenance in the category of plan expenditure D. To raise revenue for road development via foreign direct investment Your Answer : C Correct Answer : A Answer Justification : Background: The fund is utilised for the development and maintenance of National highways, State roads, Rural roads and for provision of road overbridges/under bridges and other safety features at unmanned Railway Crossings. In order to mobilise the fund, the Central Road Fund Act 2000 proposed to levy and collect by way of cess, a duty of excise and duty of customs on petrol and high speed diesel oil. Learning: The bill seeks to amend the Central Road Fund Act, 2000, to allocate 2.5% of the proceeds of CRF for development and maintenance of NWs and a reduction in the share provided for development of National Highways. Q Source: http://www.insightsonindia.com/2017/07/25/insights-daily-current-affairs-25-july-2017/

58 The Banking Regulation (Amendment) Bill, 2017 seeks to address which of these major issues in the banking sector? A. Non-performing assets B. Lack of competition in public sector banks C. Increasing participation of women in top tier management of banks D. Financial inclusion in rural areas Your Answer : A Correct Answer : A Answer Justification : Background: Stressed assets (NPAs or bad debt) in the banking system have reached unacceptably high levels and hence, urgent measures were required for their speedy resolution. Therefore, the government considered it necessary to make provisions in the Banking Regulation Act, 1949 for authorizing the Reserve Bank of India to issue directions to any banking company or banking companies to effectively use the provisions of the Insolvency and Bankruptcy Code, 2016 for timely resolution of stressed assets. Learning: The 2017 bill gives powers to the Reserve Bank of India (RBI) to ask any bank to initiate insolvency proceedings and give directions for resolution of stressed assets. The RBI would also be empowered to issue other directions for resolution, appoint or approve for appointment, authorities or committees to advise the banking companies for stressed asset resolution. Q Source: http://www.insightsonindia.com/2017/07/25/insights-daily-current-affairs-25-july-2017/ 59 In 2015, under the Indradhanush plan, the government had announced capital infusion of over seventy thousand crores in public sector banks for four years. However, credit rating agencies had pointed out that the sum was insufficient because banks needed additional funds to 1. Meet Basel-III norms 2. Tackle the menace of rising bad loans Which of the above is/are correct? A. 1 only B. 2 only C. Both 1 and 2 D. None Your Answer : C Correct Answer : C Answer Justification : Background: "Basel III" is a comprehensive set of reform measures, developed by the Basel Committee on Banking Supervision, to strengthen the regulation, supervision and risk management of the banking sector. It is an improvement over Base I and Basel II norms released much earlier. These norms may include reducing bank investment in risky assets, increasing their capital base, strengthening management etc. Justification: Statement 1: Rating agencies pointed that apart from PSBs face lack of competition and if business as usual continues, they will face dearth of funds for expansion because: Private sector will give them a tough competition in coming years Basel III norms required more funds to absorb greater risks for banks

Statement 2: The background has been explained in another question on banking amendment bill. Q Source: http://www.insightsonindia.com/2017/07/29/insights-daily-current-affairs-29-july-2017/ 60 Methanol is a promising fuel for waterways in India because 1. It does not have any smog based emissions. 2. It is cheaper than most fossil fuels. 3. India manufactures all its requirement of methanol by domestic production. Select the correct answer using the codes below. A. 1 and 2 only B. 2 and 3 only C. 2 only D. 3 only Your Answer : C Correct Answer : C Answer Justification : Justification: Across the world, methanol is emerging as a clean, sustainable transportation fuel of the future. Methanol can be blended with gasoline in low-quantities and used in existing road vehicles, or it can be used in high-proportion blends. Statement 1: Methanol is a clean-burning fuel that produces fewer smog-causing emissions such as sulphur oxides (SOx), nitrogen oxides (NOx) and particulate matter. This can improve air quality and related human health issues. Statement 2 and 3: It is clean, cheaper than fossil fuels and a good substitute for heavy fuels. India imports methanol from Saudi Arabia and Iran at present. Technology is also being commercialized to use methanol as a diesel substitute. Learning: Methanol is most commonly produced on a commercial scale from natural gas. It can also be produced from renewable sources such as biomass and recycled carbon dioxide. As a high-octane vehicle fuel, methanol offers excellent acceleration and power. It also improves vehicle efficiency. The government has asked the think-tank NITI Aayog to study the automobile standards developed in China to use methanol as an alternative fuel. Experts believe that Methanol economy will help India use its vast reserves of coal while driving import substitution. Q Source; http://www.insightsonindia.com/2017/08/01/insights-daily-current-affairs-01-aug-2017/ 61 E-RaKAM is a first-of-its-kind initiative that 1. Leverages technology to connect farmers of the smallest villages to the biggest markets of the world 2. Establishes grading and standardization centres for agricultural produce in the remotest locations of the country to promote food processing Which of the above is/are correct? A. 1 only B. 2 only C. Both 1 and 2 D. None Your Answer : B

Correct Answer : A Answer Justification : Justification: E-RaKAM is a digital initiative bringing together the farmers, FPOs, PSUs, civil supplies and buyers on a single platform to ease the selling and buying process of agricultural products. Under this initiative, e-RaKAM centres are being developed in a phased manner throughout the country to facilitate farmers for online sale of their produce. The portal is a joint initiative by state-run-auctioneer MSTC and Central Warehousing Corporation arm CRWC. Q Source: http://www.insightsonindia.com/2017/08/02/insights-daily-current-affairs-02-aug-2017/ 62 Who is a Parliamentary secretary? A. Assists a Minister in his or her duties B. Heads the Lok Sabha secretariat C. Discharges the functions of speaker when she is on leave D. In charge of convening and summoning a session of the Parliament Your Answer : A Correct Answer : A Answer Justification : Learning: A Parliament Secretary is similar to a Minister of State who assists a Minister in his or her duties. They come next to cabinet ministers and ministers of state in the hierarchy of ministerial executive. Recently there was a controversy over the 21 AAP MLAs who were appointed Parliamentary Secretaries to Ministers of the Delhi government. It raises the issue whether they should be considered as ministers, and if so, should the cap of 15% for the size of council of ministers should apply to them. Q Source: Frequently see in news 63 Consider the following statements about Liquid nitrogen, that was in news for some time. 1. It starts to boil once it reaches room temperature. 2. It is an inflammable gas. 3. It is banned for use as an additive in frozen food in India. Select the correct answer using the codes below. A. 1 only B. 2 and 3 only C. 3 only D. None of the above Your Answer : C Correct Answer : D Answer Justification : Justification: Statement 1: Liquid nitrogen, which has a boiling point of -195.8 degree Celsius, is used by molecular gastronomy chefs to instantly freeze food and drinks. Recently, it is also being popular in preparing cocktails, as it can immediately chills glasses of ingredients. It also adds a smoking effect to the drink. It cools any substance immediately. The rapidity of cooling is also responsible for smaller ice crystals which gives a smooth texture to the ice cream.

Issues: As it evaporates, liquid nitrogen freezes everything around it, including tissues that come in contact with it. According to experts, food and drinks that are prepared with liquid nitrogen should be consumed only after all the gas has bubbled off. Statement 2: It is a coloured, odourless, non-inflammable gas. It is a cryogen, which means it is used to create extremely low temperatures. The gas is so cold that it is capable of rapid freezing any living tissue coming in contact with it. Statement 3: Liquid nitrogen is permitted as an additive in frozen food as per the guidelines of the national regulatory body Food Safety and Standards Authority of India (FSSAI). However, when it comes to the use of liquid nitrogen it is a grey area. There is no clear-cut guideline for it and generally it is considered to be a novel technique, which can be used by food business operators. Q Source: http://www.insightsonindia.com/2017/08/01/insights-daily-current-affairs-01-aug-2017/ 64 Under WTO rules, a member country cannot discriminate between its trade partners. Hence 1. A country cannot enter into a free trade agreement with another country without the authorization of WTO 2. If a special status is granted to a trade partner, it should usually be extended to all members of the WTO. Which of the above is/are correct? A. 1 only B. 2 only C. Both 1 and 2 D. None Your Answer : D Correct Answer : B Answer Justification : Justification: Statement 1: Countries don’t require the permission of WTO for their trade related activities. However, they can move WTO if a country violates the rules of trade set by WTO. They are free to enter into FTAs with other countries. Statement 2: This is called as MFN status. As per a WTO rule, every member of WTO requires to accord this status to other member countries. India has already granted this status to all WTO members including Pakistan. Pakistan is yet to reciprocate. Most Favoured Nation is a treatment accorded to a trade partner to ensure nondiscriminatory trade between two countries vis-a-vis other trade partners. MFN, however, allows for countries to enter into regional trade agreements. It also relaxes such requirements in case of LDCs and other developing countries. Q Source: http://www.insightsonindia.com/2017/08/01/insights-daily-current-affairs-01-aug-2017/ 65 The objective of the GST composition scheme is to 1. Reduce duties and taxes on all raw materials to zero 2. Maximize the tax on value added to the final product 3. Reduce the tax compliance cost for small businesses 4. Charge differential taxes for the same product based on different compositions

Select the correct answer using the codes below. A. 3 only B. 1 and 2 only C. 4 only D. 1, 2 and 3 only Your Answer : C Correct Answer : A Answer Justification : Justification: The composition scheme is an alternative method of levy of tax designed for small taxpayers whose turnover is up to Rs 75 lakh — Rs 50 lakh in the case of eight north-eastern states and the hilly state of Himachal Pradesh. To promote small business, and reduce their tax burden, the scheme provides that: While a regular taxpayer has to pay taxes on a monthly basis, a composition supplier is required to file only one return and pay taxes on a quarterly basis. Also, a composition taxpayer is not required to keep detailed records that a normal taxpayer is supposed to maintain. Learning: As per the Central GST Act, businesses are eligible to opt for the composition scheme if a person is not engaged in any inter-state outward supplies of goods and not into making any supply of goods through an electronic commerce operator who is required to collect tax at source. Five lakh businesses have opted for the GST composition scheme, which allows them to pay taxes at a concessional rate and makes compliance easy. Q Source: http://www.insightsonindia.com/2017/08/01/insights-daily-current-affairs-01-aug-2017/ 66 A new representative body — US- India Strategic Partnership Forum (USISPF) — is being set up to further enhance A. Business relations between the two countries B. Defense related strategic relations C. Education and technology related exchange D. All of the above Your Answer : D Correct Answer : A Answer Justification : Learning: The non-profit corporation aims to promote bilateral trade and work on creating meaningful opportunities that have the power to change the lives of citizens. The body will work closely together with businesses and government leaders to achieve its goals of driving economic growth, job creation, innovation, inclusion and entrepreneurship. Q Source: http://www.insightsonindia.com/2017/08/02/insights-daily-current-affairs-02-aug-2017/ 67 The Ministry of Tribal Affairs is implementing the “Scheme of Development of Particularly Vulnerable Tribal Groups(PVTGs)”. The scheme focuses on which of these key parameters of development? 1. Providing regular unemployment allowances 2. Economic development of PVTGs through horticulture 3. Land distribution and land development 4. Promotion of traditional and tribal games and sports Select the correct answer using the codes below.

A. 2, 3 and 4 only B. 1 and 3 only C. 2 and 4 only D. 1, 2, 3 and 4 Your Answer : A Correct Answer : A Answer Justification : Justification: It aims at planning their socio-economic development in a comprehensive manner while retaining the culture and heritage of the community. Activities under the Scheme includes Livelihood, Employment opportunities and economic development of PVTGs through Agriculture, Horticulture, Animal Husbandry, Dairy, and Skilling/ Vocational Training etc. It further includes Education, Health (Gap filling for effective health service delivery beyond NHM etc) and Provision of safe drinking water. Land distribution, land development, Social security, Connectivity (Road and Telecommunication), Supply of Electricity and irrigation are also taken care of including sports. Housing and Habitat covers funds for housing for PVTGs, primarily through special assistance under IAY and additionally through this scheme, for gap filling and for enhancing scope of coverage. Q Source: Samir/JD/jk (Release ID :168896) 68 A microbial consortium called SONA was recently seen in news. It is related to A. Bio-remediation of oil spills B. Identifying gold mines using microbes without physical exploration C. Improving jute cultivation yield D. Treatment of sewage sites Your Answer : Correct Answer : C Answer Justification : Learning: Improved Cultivation and Advanced Retting Exercise for Jute (Jute – ICARE) was launched in 2015 to popularize/introduce some of the better agronomic practices and recently developed microbial-assisted retting among farmers. Central Research Institute for Research in Jute and Allied Fibres (CRIJAF) has also recently developed a microbial consortium called SONA, to enhance the quantity of fibre yield by 20% as well as its quality in terms of grade by at least 1 ½ grades. Also, under the project, regular SMSes are sent in regional languages on improved practices in jute cultivation, to registered farmers. Q Source: PIB: No release ID: Ministry of Textiles: 25 July, 2017 69 The Union Cabinet has given its ex-post facto approval for amendment of the Constitution (Application to Jammu & Kashmir) Order, 1954 by way of the Constitution (Application to Jammu & Kashmir) Amendment Order, 2017. This was done in order to A. Apply GST regime in the State of Jammu & Kashmir B. Allow alternative communication models like prepaid sim cards in J&K that were earlier blocked due to security concerns C. Provide a separate flag to the state of J&K

D. Assign a permanent advisor from the Union government to the state of J&K Your Answer : A Correct Answer : A Learning: An ordinary amendment to the constitution does not apply necessarily to the state of J&K which has a separate constitution of its own. The President needs to give approval to certain amendments about which you will read later in Laxmikanth. The approval paves the way for applicability of Goods and Services Tax regime in the State of Jammu & Kashmir. Q Source: AKT/VBA/SH (Release ID :169110) 70 The Indian Council for Cultural Relations (ICCR has several Indian Cultural Centres abroad. Consider the following about them. 1. These cultural centres showcase India’s soft power. 2. There is no Indian Cultural Centre in Central Asia and the Middle east. 3. Activities in these centres are organized with the help of involvement of local community and Indian Diaspora. Select the correct answer using the codes below. A. 1 and 3 only B. 2 only C. 2 and 3 only D. 1 only Your Answer : A Correct Answer : A Answer Justification : Justification: Statement 1: Soft power of India includes its culture, diaspora, ancient heritage like yoga and the skills of its population. Statement 2: During the recent visit of Prime Minister of India to Israel, announcement for the opening of Indian Cultural Centre in Israel was made. There are centres in Central Asian countries like Kazakhstan. Russia is another centre, but it is not counted in central Asia. Statement 3: Cultural Centres overseas are ICCR’s important arms, which play significant role in promotion of India’s soft power strength through its variety of activities like facilitating teaching of Indian dance, music, yoga, Hindi etc., organizing cultural performances etc. You can find the list of all such centres at the Q Source. Q Source: YSK (Release ID :169048) 71 The Amasr Act, 2010 that was recently seen in news concerns A. Preservation of monuments B. Immigration and refugees in India C. Genetic cleansing of livestock population D. Prohibited chemicals and weapons Your Answer : Correct Answer : A Answer Justification : Learning: The proposal for amendment of the Amasr Act, 2010 has been approved by the Cabinet

and the pertaining Bill has been moved to Lok Sabha. The proposal is to allow public works or projects essential for public in prohibited area within 100 meter from protected monument but not having substantial impact on preservation, safety, security or access to the monument or its immediate surrounding including visual ambiance. Q Source: Sanjay Kumar/jk/Parl. No. 1/26-07-2017 (Release ID :169062) 72 Private Entrepreneurs Guarantee (PEG) Scheme concerns A. Manufacturing MSMEs B. Warehousing capacity for food storage C. Venture Capital Funds that have been established by the Government D. Guaranteed Land acquisition for mega projects Your Answer : A Correct Answer : B Answer Justification : Learning: Under PEG, storage capacity is created by private parties, Central Warehousing Corporation (CWC) and State Agencies for guaranteed hiring by FCI. This is because the state alone cannot provide such facilities at all locations. It makes economic sense for private players to finance the construction of these warehouses and then lease it out for rent. Q Source: www.dfpd.nic.in/peg.htm 73 Which of these nations share closest latitudes? #46460 A. Aswan, Riyadh and Abu Dhabi B. Cairo, Istanbul and Damascus C. Tehran, Ashgabat and Aleppo D. Mosul, Muscat and Tel-Aviv Your Answer : Correct Answer : A Answer Justification : Justification: Closest latitudes mean that they are located in a narrow band of latitude. They can be located at different longitudes. Q Source: Map based questions: West Asia 74 Consider the following with reference to rubber production in India. 1. Kerala is the largest producer of natural rubber in India. 2. The government has deregulated synthetic rubber industry in India. 3. As of date, no dedicated scheme or policy has been enacted to develop rubber sector in India. Select the correct answer using the codes below. A. 1 only B. 2 and 3 only C. 1 and 2 only D. 1, 2 and 3 Correct Answer : C Answer Justification : Justification: Statement 1: India is the third largest natural rubber producing country of the world, next to Thailand and Indonesia, producing about 9 per cent of the global output.

Kerala is the largest producer, followed by TN. Statement 2: So far as artificial/synthetic rubber is concerned, Chemical & Petrochemical Industry is delicensed and decontrolled and Government acts as facilitator in this sector. Statement 3: In order to increase production of Natural Rubber in the country the Government through the Rubber Board is implementing the scheme “Sustainable and Inclusive Development of Natural Rubber Sector” wherein support is provided for plantation development and extension, strengthening research, technology upgradation and market development, Human resource development etc. Q Source: MJPS (Release ID :169314) 75 Consider the following matches of periods with the beginning of major activities in the history of human civilization: 1. Building of megaliths: 3000 years ago 2. Cotton cultivation: 2000 years ago 3. Domestication of animals: 12000 years ago Select the correct answer using the codes below. A. 1 and 2 only B. 1 and 3 only C. 2 only D. 1, 2 and 3 Your Answer : D Correct Answer : B Answer Justification : Justification: Statement 1: Beginning of cities started about 4700 years ago. Settlement at Inamgaon began between 3600 and 2700 years ago. Statement 2: This started at Mehrgarh about 7000 years ago. Statement 3: Domestication was a gradual process that took place in many parts of the world. It began about 12,000 years ago. Some of the earliest plants to be domesticated were wheat and barley. The earliest domesticated animals include sheep and goat. Q Source: General: Chapter 1: Themes in Indian History: Part -I 76 Which of these strive to ensure equity among Indian citizens? 1. Constitution of India 2. Laws enacted by the Parliament 3. Schemes and programmes of the government Select the correct answer using the codes below. A. 1 only B. 2 and 3 only C. 1 and 3 only D. 1, 2 and 3 Your Answer : A Correct Answer : D Answer Justification : Justification: Statement 1: Fundamental rights ensure right to equality and remove arbitrary discrimination. DPSP strives to create a equitable society through special provisions for weaker

sections. Statement 2: Laws like SC/ST Atrocities act, Forest Rights Act ensure justice to weaker and marginalized sections. Statement 3: MGNREGA, Food security Act etc. ensure that left behind sections are provided with livelihood, food etc. Q Source: Page 10: Social and Political Life: 7th NCERT 77 The first state in India to introduce the Mid Day Meal (MDM) scheme was A. Kerala B. Gujarat C. Tamilnadu D. Bihar Your Answer : C Correct Answer : C Answer Justification : Learning: Other than the TN model of MDM, in 2001, the Supreme Court asked all state governments to begin this programme in their schools within six months. Later, it was made a central programme that today feeds more than 12 crore children all over India. The Midday Meal Scheme is covered by the National Food Security Act, 2013. The central and state governments share the cost of the Midday Meal Scheme, with the centre providing 60 percent and the states 40 percent Q Source: Page 11: Social and Political Life: 7th NCERT 78 The United Nations Children's Fund (UNICEF) is a United Nations (UN) programme that provides humanitarian and developmental assistance to children and mothers in developing countries. Consider the following with reference to UNICEF’s association with India. 1. UNICEF provided equipment and technical assistance for the first Penicillin Plant established in India. 2. Within a decade of independence, UNICEF signed an agreement with the Government of India to fund some cooperative milk processing plants to support white revolution. 3. UNICEF helped India in developing the India Mark II which is now the world’s most widely used handpump. Select the correct answer using the codes below. A. 1 only B. 2 only C. 3 only D. 1, 2 and 3 Your Answer : B Correct Answer : D Answer Justification : Justification: Statement 3: During the 1970s, UNICEF became a key partner with the Government of India in the world’s largest rural water supply programme. UNICEF brought drilling rigs to India which could drill boreholes in hard rock. The Government supplied handpumps. But, due to some problems, India mark II was later adopted. Learning: Gender issues were mainstreamed into the training and communication strategy for the

2011 Census with the help of UNESCO. The Government, in partnership with UNICEF, WHO, the Bill & Melinda Gates Foundation, Rotary International and the Centers for Disease Control and Prevention contributed to almost universal awareness of the need to vaccinate all children under five against polio. As a result of these efforts, India was removed from the list of endemic countries in 2014. Q Source: Additional Research: Page 25: Social and Political Life: 7th NCERT http://unicef.in/WhoWeAre/History 79 Archaeological Survey of India was founded by A. Alexander Cunningham B. Joseph Barruel C. Lord Lawrence D. Mortimer Wheeler Your Answer : A Correct Answer : A Answer Justification : Learning: ASI is attached to the Ministry of Culture that is responsible for archaeological research and the conservation and preservation of cultural monuments in the country. Cunningham carried out a detailed survey of the Buddhist monuments which lasted for over half a century. While he funded many of his early excavations himself, in the long run, he realised the need for a permanent body to oversee archaeological excavations Q Source: Additional Research: Page 6: Themes in Indian History – Part I 80 Which of these sovereign states do NOT have armed forces? 1. Costa Rica 2. Solomon Islands 3. Madagascar Select the correct answer using the codes below. A. 1 and 2 only B. 3 only C. 1 and 3 only D. 2 only Your Answer : Correct Answer : A Answer Justification : Justification: There are 23 countries that have no active military force, including Costa Rica, Iceland, Panama, Micronesia, the Marshall Islands, and The Vatican. These nations vary in size, history, and reasons behind choosing to not have a standing army. Statement 1: Not having an army helped the Costa Rican government to spend the money that the army would have used, on health, education and other basic needs of the people. Q Source: Additional Research: Page 28: Social and Political Life: 7th NCERT 81 That the State should “raise the level of nutrition and the standard of living and to improve public health” is a provision under A. Fundamental right B. Directive Principle of State policy

C. Fundamental Duty D. Legal rights Your Answer : B Correct Answer : B Answer Justification : Learning: Right to Health is not included as an explicit fundamental right in the Indian Constitution. Most provisions related to health are under DPSP. These are: Article 38 says that the state will secure a social order for the promotion of welfare of the people. Providing affordable healthcare is one of the ways to promote welfare. Article 39(e) calls the state to make sure that health and strength of workers, men and women, and the tender age of children are not abused. Article 41 imposes duty on state to provide public assistance in cases of unemployment, old age, sickness and disablement etc. Article 42 makes provision to protect the health of infant and mother by maternity benefit. Article 47 make it duty of the state to improve public health, securing of justice, human condition of works, extension of sickness, old age, disablement and maternity benefits and also contemplated. Further, State’s duty includes prohibition of consumption of intoxicating drinking and drugs are injurious to health. Article 48A ensures that State shall Endeavour to protect and impose the pollution free environment for good health. Q Source: Page 29: Social and Political Life: 7th NCERT 82 Which of these nations has nationalized the Suez Canal? A. Egypt B. Syria C. Israel D. Saudi Arabia Your Answer : A Correct Answer : A Answer Justification : Learning: The Suez Canal is considered to be the shortest link between the east and the west due to its unique geographic location. It is an important international navigation canal linking between the Mediterranean sea at Port said and the red sea at Suez. Egypt nationalized the canal in 1956. Q Source: Map based questions: West Asia 83 Consider the following statements. 1. A Governor is the Head of a State of India, whereas President is the head of the Indian state. 2. Governors are appointed by the President of India and he is free to consult the state chief minister in this matter. Which of the above is/are correct? A. 1 only B. 2 only

C. Both 1 and 2 D. None Your Answer : D Correct Answer : C Answer Justification : Justification: Statement 1: The Head (ceremonial) of the State is the Governor. Head of government is the Chief Minister. She/He is appointed by the Central Government to ensure that the State Government works within the rules and regulations of the Constitution. Statement 2: The President may nor may not consult the CM, but it is a convention to do so since both need to work in tandem. Q Source: Page 33: Social and Political Life: 7th NCERT 84 An Internal Complaints Committee (ICC) is usually constituted at workplaces according to which of these acts? A. Consumer Protection Act, 1984 B. Factories Act, 1948 C. Sexual Harassment of Women at Workplace (Prevention, Prohibition and Redressal) Act, 2013 D. None of the above Your Answer : Correct Answer : C Answer Justification : Learning: Under the Act, which also covers students in schools and colleges as well as patients in hospitals, employers and local authorities will have to set up grievance committees to investigate all complaints. These grievance committees are known as ICC. The District Officer is required to constitute a Local Complaints Committee at each district, and if required at the block level. The Complaints Committees have the powers of civil courts for gathering evidence. Q Source: Additional Research: Page 64: Social and Political Life: 7th NCERT 85 The Advertising Standards Council of India (ASCI) is a 1. Self-regulatory institution 2. Non-government body 3. Body that can prohibit certain ads from appearing on TV Select the correct answer using the codes below. A. 1 and 2 only B. 3 only C. 2 only D. 1, 2 and 3 Your Answer : B Correct Answer : D Answer Justification : Concept & Background: When an advertiser is creating an ad, the consumer is his audience. If a consumer feels that a particular advertisement is in bad taste or is false in its claims, they

need a body or council to whom they can air their grievances and who will take any appropriate action, if necessary. ASCI as a self-regulatory body governing advertising content is the ideal medium as its purpose is to serve both the advertisers as well as the consumers. There is no other non governmental body in India which regulates the advertising content that is released in India. Justification: If an ad that is released in India seems objectionable, a person can write to ASCI with their complaint. This complaint will be deliberated on by the complains council after providing due process to advertiser to defend the ad against the complaint. If complaint is upheld then the ad is voluntarily either withdrawn or modified. In 2007, the Government of India amended the Cable TV Network Rules’ Advertising Code by which ads which violate ASCI code cannot be permitted on TV. Q Source: Additional Research: Page 83: Social and Political Life: 7th NCERT 86 Which of these nations border both Caspian Sea and Black Sea? A. Iran B. Turkey C. Russia D. Georgia Your Answer : B Correct Answer : C Answer Justification : Learning: Q Source: Map based questions: West Asia 87 Consider the following statements. 1. Every state in India has a Legislative Assembly. 2. The strength of legislative assembly in a state is fixed by the President. Which of the above is/are correct? A. 1 only B. 2 only C. Both 1 and 2 D. None Your Answer : A Correct Answer : A Answer Justification : Justification: Statement 1: But, all UTs do not have a legislative assembly. Only, Delhi and Puducherry have it. Statement 2: This is based on the population of the state. A state with a larger population like UP will have more seats in the assembly. Q Source: Page 32: Social and Political Life: 7th NCERT 88 Nations that border the Persian Gulf are 1. Saudi Arabia 2. United Arab Emirates 3. Iran

4. Kuwait 5. Jordan Select the correct answer using the codes below. A. 2 and 4 only B. 1, 2, 3 and 4 only C. 3 and 5 only D. 1 and 5 only Your Answer : Correct Answer : B Answer Justification : Justification: Q Source: Map based questions: West Asia 89 What are the properties of X-rays? 1. X-rays have much longer wavelengths than visible light. 2. X-ray photons carry enough energy to ionize atoms and disrupt molecular bonds. 3. X-rays do not interact with matter at all. Select the correct answer using the codes below. A. 1 and 2 only B. 2 only C. 3 only D. 1, 2 and 3 Your Answer : D Correct Answer : B Answer Justification : Justification: Statement 1: They have shorter wavelengths and higher frequencies. This makes it possible to probe structures much smaller than can be seen using a normal microscope. This property is used in X-ray microscopy to acquire high resolution images, and also in X-ray crystallography to determine the positions of atoms in crystals. Statement 2: This makes it a type of ionizing radiation, and therefore harmful to living tissue. A very high radiation dose over a short period of time causes radiation sickness, while lower doses can give an increased risk of radiation-induced cancer. Statement 3: X-rays interact with matter in three main ways, through photoabsorption, Compton scattering, and Rayleigh scattering. The strength of these interactions depends on the energy of the X-rays and the elemental composition of the material. We will try covering these separately in upcoming tests because they are important. Q Source: Additional Research: Page 25: Social and Political Life: 7th NCERT 90 Consider the following statements. 1. Dasht-e Kavir is a great salt desert located in Arabian Peninsula. 2. Dasht-e Lut is a large desert located in Iran that has been inscribed on UNESCO's World Heritage List. Which of the above is/are correct? A. 1 only

B. 2 only C. Both 1 and 2 D. None Your Answer : D Correct Answer : B Answer Justification : Justification: Statement 1: Also known as Kavir-e Namak (literally 'salty lowlands') and the Great Salt Desert, it is a large desert lying in the middle of the Iranian plateau. Statement 2: The Lut Desert, widely referred to as Dasht-e Lut is a large salt desert located in the provinces of Kerman and Sistan and Baluchestan, Iran. It is the world's 27th-largest desert, and was inscribed on UNESCO's World Heritage List in 2016. The surface of its sand has been measured at temperatures as high as 70 °C making it one of the world's driest and hottest places. Q Source: Map based questions: West Asia 91 Harappans were aware of and practiced which of these number systems in their weights and measures? A. Binary system B. Decimal system C. Hexadecimal system D. Both (a) and (b) Your Answer : D Correct Answer : D Answer Justification : Learning: Exchanges were regulated by a precise system of weights, usually made of a stone called chert and generally cubical with no markings. The lower denominations of weights were binary (1, 2, 4, 8, 16, 32, etc). While the higher denominations followed the decimal system, the smaller weights were probably used for weighing jewellery and beads and bigger weights were used for food grains. Q Source: Additional Research: Page 15: Themes in Indian History – I 92 Among the earliest chalcolithic cultures in India, the Ahar or Banas culture was discovered in the A. Mewar region of Rajasthan B. Gwalior region of Madhya Pradesh C. Bundelkhand region of Uttar Pradesh D. Kangra region of Himachal Pradesh Your Answer : A Correct Answer : A Answer Justification : Learning: Nearly one hundred sites of the culture have been located along its principal axis, i.e., the valleys of river Banas and its tributaries and subtributaries in Banswara, Udaipur etc. The technology at Ahar was based mainly on copper and very few microblades and microliths have been discovered. Ahars were a separate culture from GJs. Located in northeast Rajasthan, the Ganeshwar-Jodhpura complex which was an early centre of agriculture and copper metallurgy in the subcontinent.

Q Source: Additional Research: Page 12: Themes in Indian History – I 93 Consider the following statements. 1. Most Ashokan inscriptions were in the Greek language while those in the northwest of India subcontinent were in Aramaic and Prakrit. 2. Ashokan inscriptions were written in both Prakrit and Brahmi scripts. Which of the above is/are correct? A. 1 only B. 2 only C. Both 1 and 2 D. None Your Answer : B Correct Answer : B Answer Justification : Justification: Statement 1: A large number of them were in Prakrit, but in the NW direction one could find Aramaic and Greek languages. Statement 2: The Aramaic and Greek scripts were used for inscriptions in Afghanistan. James Prinsep, an officer in the mint of the East India Company, deciphered Brahmi and Kharosthi, two scripts used in the earliest inscriptions and coins. This gave a new direction to investigations into early Indian political history. Q Source: Page 28: Themes in Indian History – I 94 With reference to Matsays, one of the early states, consider the following statements. 1. They were one of the Indo-Aryan tribes of Vedic India. 2. Their rule extended to the whole of eastern and central India. 3. They were an appendage of Ashmaka dynasty. Select the correct answer using the codes below. A. 1 only B. 2 and 3 only C. 3 only D. 1, 2 and 3 Your Answer : A Correct Answer : A Answer Justification : Justification: Statement 1 and 2: By the late Vedic period, they ruled a kingdom located south of the Kurus, and west of the Yamuna River which separated it from the kingdom of the Panchalas. It roughly corresponded to the former state of Jaipur in Rajasthan. Statement 3: The capital of Matsya was at Viratanagari (present-day Bairat). Alwar has been a part of Matsya region. Ahmakas were further down south, see map below 95 In Ancient India, a means of claiming high status for the Kings was to identify with a variety of deities. Consider the following with reference to it. 1. Kushana rulers installed god-like colossal statues of them. 2. Some Kushanas adopted the title devaputra which implied the “son of the God”. 3. Vijayanagara kings claimed to rule on behalf of the god Virupaksha. 4. Vijayanagara rulers adopted the title ‘brahma-varna” to segment themselves as the highest castes equivalent to God.

Select the correct answer using the codes below. A. 1, 2 and 3 only B. 2 and 4 only C. 3 only D. 1, 2, 3 and 4 Your Answer : Correct Answer : A Answer Justification : Justification: Statement 1: Huge statues of Kushanas have been found installed in a shrine at Mat near Mathura (Uttar Pradesh). Similar statues have been found in a shrine in Afghanistan as well. Statement 2: The title devaputra was possibly inspired by Chinese rulers who called themselves sons of heaven. Statement 3: They indicated their close links with the gods by using the title “Hindu Suratrana” meaning Hindu Sultan. Q Source: Additional Research: Page 36: Themes in Indian History – I 96 The institution of Samanta finds mention for the first time in epigraphs of northern India dating to the 6th century. It is most closely associated with the origin and growth of which of these systems in India? A. Feudalism B. Bonded Labour C. State ownership of land D. Standing army system Your Answer : A Correct Answer : A Answer Justification : Justification: Options B, C and D existed much prior to 6th century, so they can’t be the answer. The institution is considered to belong properly to the Gupta Empire, however, some vague evidence is found in South India as well. The Samanta in South-India was used to mean a vassal to an emperor. In North-India, the earliest use of the term in a similar sense was in Bengal where someone was described as Samanta-Chudamanih (best among feudatories) of the imperial Guptas. The Samanta vassal provided military support to the Monarch and governed over a portion of a territory. Learning: The term Indian feudalism is used to describe zamindar, jagirdar, sardar, mankari, deshmukh, chaudhary and samanta. In this system, a nobility gave the emperor military services in exchange of land. There was a separate class of labourers or peasants who were provided military protection in exchange of labour and share of their produce. They were allowed to live on the lord’s land. Q Source: Additional Research: Themes in Indian History – I 97 In the Indian subcontinent, the first coins to bear the names and images of rulers were issued by the A. Indo - Greeks B. Mauryas C. Parthians D. Yaudheyas

Your Answer : A Correct Answer : A Answer Justification : Learning: Mauryas issued punch marked coins made of silver and copper. Indo-greeks used names of rulers on coins. First gold coins were issued by Kushanas (some sources dispute this fact). Coins were also issued by tribal republics of Punjab and Haryana called the Yaudheyas. Archaeologists have unearthed several thousand copper coins issued by the Yaudheyas. Some of the most spectacular gold coins were issued by the Gupta rulers. Q Source: Page 44: Themes in Indian History – I 98 The earliest inscriptions recording royal land grants were issued during A. Guptas B. Satvahanas C. Pallavas D. Hoysalas Your Answer : A Correct Answer : B Answer Justification : Learning: In early India, land possessed great value and the gift of an estate was a marker of status. While the earliest inscriptions recording royal land grants were issued during Satavahana rule, the practice truly grew in scope from the fourth century CE. By the 5th-6th centuries, ruling dynasties across the subcontinent, as well as their subordinates and feudatories were all engaged in making such grants. Land grant charters were usually engraved on sheets of hammered copper. They are described as tamrapatta, a ‘plate of copper’ or tamrashasana, ‘an order in copper’. Q Source: Page 50: Themes in Indian History – I 99 What is a catcher beach? A. A beach with low shorelines B. A tidal estuary with no opening mouth C. A place where marine debris tends to pile up or aggregate D. A beach devoid of mangroves and corals Your Answer : Correct Answer : C Answer Justification : Learning: A catcher beach should not be confused with a dumping ground or heavily trashed public beach. A catcher beach typically receives its accumulations of debris due to its shape and location in combination with high-energy waves, storms, or winds. A specific example of a catcher beach can be found along the shores of Gore Point, Alaska. The geography of this location makes it a very high-density catcher beach, as it sticks out like a hook into the Gulf of Alaska current. Q Source: Surprise questions 100 The Centre has banned the manufacture and import of Polychlorinated Biphenyls (PCBs). This is

because A. It is a toxic pollutant. B. It is responsible for coral bleaches across the eastern coast of India. C. It was being oversupplied in the market leading to price crashes and poor producer margins. D. It was heavily used by farmers as an additive in their NPK fertilizers. Your Answer : A Correct Answer : A Answer Justification : Learning: PCBs are synthetic organic chemicals used in many different products including electrical equipment, inks, adhesives, flame-retardants, and paints. Its production and use are now severely restricted in many countries because of possible impacts on human health and the environment. The use of PCBs in any form shall be completely prohibited by 2025. The government has directed users not to drain or discharge PCBs directly or indirectly on land, in surface water or from effluent treatment plants. The government's decision is in line with the Stockholm Convention, under which signatory countries are to prohibit or take necessary legal and administrative measures to eliminate the production and use of PCBs.

View more...

Comments

Copyright ©2017 KUPDF Inc.
SUPPORT KUPDF